If f(x) is an exponential function where f(3) = 11 and f(5.5) =57, then find the value of f(10.4), to the nearest hundredth

Answers

Answer 1

Solving a system of equations we can find the exponential function, then we can evaluate it to get:

f(10.4)= 1,427.2

How to find the value of f(10.4)?

We know that f(x) is a exponential function, then we can write it as:

f(x) = A*b^x

First, we know that:

f(3) = 11

f(5.5) = 57

Then we can write a system of equations:

11 = A*b^3

57 = A*b^5.5

Taking the quotient between the two equations we will get:

57/11 = (A*b^5.5)/(A*b^3)

5.18 = b^(5.5 - 3)

5.18 = b^2.5

Solving for b we will get:

(5.18)^(1/2.5) = b

1.93 = b

with that, we can get the value of b.

11 = A*(1.93)^3

A = 11/(1.93)^3

A = 1.53

So the function is:

f(x)= 1.53*(1.93)^x

Then:

f(10.4) = 1.53*(1.93)^10.4 = 1,427.2

Learn more about exponential functions at:

https://brainly.com/question/2456547

#SPJ1


Related Questions

a right triangle has a hypotenuse of length 7 inches. If one angle is 38 degrees, find the length of each leg.

Answers

Check the picture below.

[tex]\sin(38^o )=\cfrac{\stackrel{opposite}{x}}{\underset{hypotenuse}{7}}\implies 7\sin(38^o)=x\implies 4.31\approx x \\\\[-0.35em] ~\dotfill\\\\ \cos(38^o )=\cfrac{\stackrel{adjacent}{y}}{\underset{hypotenuse}{7}}\implies 7\cos(38^o)=y\implies 5.52\approx y[/tex]

Make sure your calculator is in Degree mode.

Answer:

4.31 in5.52 in

Step-by-step explanation:

You want the leg measures in a right triangle with a hypotenuse of 7 inches and an angle of 38°.

Trig functions

The mnemonic SOH CAH TOA reminds you of the relations between side lengths and trig functions:

  Sin = Opposite/Hypotenuse   ⇒   Opposite = Hypotenuse×Sin

  Cos = Adjacent/Hypotenuse   ⇒   Adjacent = Hypotenuse×Cos

Application

The given triangle will have opposite and adjacent sides of ...

  opposite = (7 in)sin(38°) ≈ 4.31 in

  adjacent = (7 in)cos(38°) = 5.52 in

The leg lengths of the triangle are 4.31 inches and 5.52 inches.

On a given planet, the weight of an object varies directly with the mass of the object. Suppose that an object whose mass is 2 kg weighs 4 N. Calculate the mass of another object that weights 18 N.

Answers

Answer:

Let m be the mass of the object and w be the weight of the object. According to the problem, weight varies directly with mass, so we can write:

w = k * m

where k is the constant of proportionality.

To find k, we can use the information given in the problem. We know that when m = 2 kg, w = 4 N. Substituting these values into the equation above, we get:

4 N = k * 2 kg

Solving for k, we get:

k = 4 N / 2 kg = 2 N/kg

Now we can use this value of k to find the mass of an object that weighs 18 N. We can rearrange the equation above to solve for m:

m = w / k

Substituting w = 18 N and k = 2 N/kg, we get:

m = 18 N / 2 N/kg = 9 kg

Therefore, the mass of the object that weighs 18 N is 9 kg.

Alberto gets a fish tank

Answers

Jackson is on point. Of all fish, catfish make up one-third of the total.

How much gravel does Alberto add to the tank in pounds?

The United States of America, Great Britain, and other English-speaking countries predominately utilise the pound as a unit of weight.

Any quantity associated with something is assumed to contain one pound if it does. The pound is the most widely used weight unit in the English-speaking world, including Britain, America, and other nations.

Zebra danios make up half of all the fish.

More fancy guppies than catfish, by a factor of two.

x = guppies

2x = catfish

x + 2x = 1/2 of the fish population

3x = 1/2 of the fish population

3x * 2 = 6x

x = 1/6 number of fancy guppies

catfish:

2x = 2(1/6) = 2/6 = 1/3 catfish.

Jackson is on point. Of all fish, catfish make up one-third of the total.

To know more about pounds, visit:

brainly.com/question/498964

#SPJ1

The complete question is,

Alberto puts 3 kinds of fish in his fish tank: zebra danios, fancy guppies, and cat fish. Half of Alberto's fish are zebra danios. He has 2 times as many catfish as he has fancy guppies. Alberto's friend Jackson says 1/3 of Alberto's fish are catfish. Is Jackson correct?

A machine in a factory makes chairs at rate of two chairs every 10 minutes how much time does the machine take to make five chairs how many minutes would it take for the factory to fulfill an order for 32 chairs show your work or explain how you determine your answers

Answers

If the machine makes 2 chairs every 10 minutes, then it makes 1 chair in 5 minutes (since 10/2 = 5).

To make 5 chairs, the machine will need 5 times as much time, which is 5 x 5 = 25 minutes.

To fulfill an order for 32 chairs, the machine will need to make 32/2 = 16 sets of 2 chairs.

This means the machine will take 16 x 10 = 160 minutes to make 32 chairs.

I obtained these answers by using basic math. First, I found out how many chairs the machine makes in one minute by dividing the rate of production (2 chairs every 10 minutes) by the number of minutes (10). That gave me the production rate of 1 chair every 5 minutes.

Using that production rate, I could then multiply it by the number of chairs I needed to find out how many minutes it would take for the machine to produce them.

For the order of 32 chairs, I used the production rate to figure out how many sets of two chairs the machine would need to make to reach the total amount. And then I multiplied that number by the time it takes to make two chairs (10 minutes) to get the total time it would take to produce 32 chairs.

HELLO PLEASE HELP!
i only need help with the second problem which is:

5. For 0≤x≤ 6, express g(x) in terms of x. Do not include +C in your final answer.

please and thanks!​

Answers

From the calculation, g increases on the interval [-3, 6) and for 0 ≤ x ≤ 6, g(x) = 6 - 1/2(x-2)²

What is a mathematical expression?

A mathematical expression is a phrase that includes at least two numbers or variables, at least one arithmetic operation, and the expression itself. This mathematical operation may be addition, subtraction, multiplication, or division. An expression's basic components are as follows: The formula is (Number/Variable, Math Operator, Number/Variable).

Since f(x) is a piecewise-defined function, we need to consider each interval separately.

For -3 ≤ x < 0, f(x) = 3, so g'(x) = 3, which is positive.

For 0 ≤ x ≤ 6, f(x) = -x + 3, which is a decreasing function, so g'(x) is also decreasing. However, since f(x) is always non-negative on this interval, g'(x) is non-negative as well.

For 6 < x ≤ 9, f(x) = -3, so g'(x) = -3, which is negative.

Therefore, g is increasing on the interval [-3, 6).

To justify this, note that g'(x) = 0 at x = 0 and x = 6, where g has local maxima. This means that g is increasing on the intervals (-3, 0) and (0, 6) and decreasing on (6, 9]. Since g is continuous, it cannot have any jumps, so it must be increasing or decreasing on each of these intervals. Since g(-3) = 0 and g(6) = 9, we know that g is increasing on the interval [-3, 6).

We can evaluate g(x) on the interval [0, 6] by integrating f(x) with respect to t from -2 to x:

g(x) = ∫_{-2}^{x} f(t) dt

On the interval [-3, 0), f(t) = 3, so we have

g(x) = ∫_{-2}^{0} 3 dt + ∫_{0}^{x} (-t + 3) dt

Simplifying the integrals, we get:

g(x) = 6 - 1/2(x-2)^2, for 0 ≤ x ≤ 6

Therefore, for 0 ≤ x ≤ 6, g(x) = 6 - 1/2(x-2)²

Learn more about expression

brainly.com/question/14083225

#SPJ1

The expression of g in terms of x  is increases on the interval [-3, 6) and for 0 ≤ x ≤ 6 is  g(x) = [tex]6 - \frac{1}{2(x-2)^2}[/tex].

What is a mathematical expression?

Mathematical expressions consist of at least two numbers or variables, at least one arithmetic operation, and a statement. It's possible to multiply, divide, add, or subtract with this mathematical operation. Unknown variables, integers, and arithmetic operators are the components of an algebraic expression. There are no symbols for equality or inequality in it.

Since f(x) is a piecewise-defined function, we need to consider each interval separately.

For -3 ≤ x < 0, f(x) = 3, so g'(x) = 3, which is positive.

For 0 ≤ x ≤ 6, f(x) = -x + 3, which is a decreasing function, so g'(x) is also decreasing.

However, since f(x) is always non-negative on this interval, g'(x) is non-negative as well.

For 6 < x ≤ 9, f(x) = -3, so g'(x) = -3, which is negative.

Therefore, g is increasing on the interval [-3, 6).

We can evaluate g(x) on the interval [0, 6] by integrating f(x) with respect to t from -2 to x:

[tex]g(x) = \int{-2}^{x} f(t) dt[/tex]

On the interval [-3, 0), f(t) = 3, so we have

[tex]g(x) = \int_{-2}^{0} 3 dt + \int_{0}^{x} (-t + 3) dt[/tex]

Simplifying the integrals, we get:

[tex]g(x) = 6 - \frac{1}{2(x-2)^2}[/tex], for 0 ≤ x ≤ 6

Therefore, for 0 ≤ x ≤ 6, g(x) = 6 - 1/2(x-2)².

To Learn more about expression refer the below link

https://brainly.com/question/112703

#SPJ1

A principal of $1500 is invested at 8.5% interest, compounded annually.

How much will the investment be worth after 14 years?

Use the calculator provided and round your answer to the nearest dollar.

Answers

The investment will be worth approximately $4468 after 14 years.

What is annual interest rate?

Annual interest rate is the rate at which an investment or loan grows in value over the course of a year, expressed as a percentage. It represents the cost of borrowing or the return on investment over one year.

According to question:

The following equation can be used to determine the future value of an investment using compound interest:

FV = P * (r/n + 1)(n*t)

Where:

FV is the investment's future value.

The principal is P. (initial investment)

The annual interest rate is represented by r,

The number of times the interest is compounded annually by n (expressed as a decimal).

t is the time (in years)

In this case:

P = $1500

r = 8.5% = 0.085

n = 1 (compounded annually)

t = 14

With these values entered into the formula, we obtain:

FV = 1500 * [tex](1 + 0.085/1)^(1*14)[/tex]

     = $4467.56

Therefore, the investment will be worth approximately $4468 after 14 years.

To know more about annual interest rate visit:

https://brainly.com/question/30573341

#SPJ1

Use the following number line to choose the correct statement. R x P > S S > Q × R Q × R > S

Answers

Answer:

Looking at the number line, we can see that R is to the left of P, and S is to the right of P. Also, Q is to the left of R, and S is to the right of Q.

So, the first statement "R x P > S" is not true, because S is to the right of both R and P.

The second statement "S > Q x R" is also not true, because Q is to the left of R, and S is to the right of both Q and R.

The third statement "Q x R > S" is true, because Q is to the left of R, and S is to the right of both Q and R. Therefore, the correct statement is:

Q x R > S

Write a fraction that has a value greater than 3/8 using 3 as the numerator

Answers

Answer: 3/4

Step-by-step explanation:

Any number as the denominator of the fraction that is less than 8 (the denominator) will give you a fraction greater than 3/8.

Another different answer could be 3/5 or 3/6.

A fraction that has a value greater than 3/8 using 3 as the numerator is 3/6, 3/4, 3/2 and so on.

What is the fraction?

In Mathematics, fractions are represented as a numerical value, which defines a part of a whole. A fraction can be a portion or section of any quantity out of a whole, where the whole can be any number, a specific value, or a thing.

The given fraction is 3/8.

Here, 3/8 = 0.375

We need write the fraction which is greater than 3/8 using 3 as the numerator is 3/6 = 0.5

Therefore, a fraction that has a value greater than 3/8 using 3 as the numerator is 3/6, 3/4, 3/2 and so on.

To learn more about the fraction visit:

brainly.com/question/1301963.

#SPJ2

The sum of 2 vector forces is <5, -3>. What is the magnitude of the resulting force?

Answers

According to the question the magnitude of the resulting force is 5.83.

What is magnitude?

Magnitude is a measure of the size or intensity of a physical quantity. It is an expression of how large or small a quantity is in comparison to a reference value. Magnitude is typically used in physics and astronomy, but it can also be used in other areas such as engineering and seismology. Magnitude is not an absolute measurement; rather, it is a relative measure of how much larger or smaller one quantity is compared to another. For example, the magnitude of a star's brightness is a measure of how much brighter it is compared to other stars.

The magnitude of the resulting force can be calculated using the Pythagorean theorem. The formula for calculating the magnitude of a vector is:
[tex]\sqrt[]{(x2 + y2)}[/tex]
In this case, x = 5 and y = -3, which gives us:
[tex]\sqrt{(52 + (-3)2)}[/tex] = [tex]\sqrt{(25 + 9)}[/tex] = √[tex]\sqrt{(34)}[/tex] = 5.83.
Therefore, the magnitude of the resulting force is 5.83.

To learn more about magnitude
https://brainly.com/question/28047791
#SPJ1

How do I solve this problem?

Answers

The measure of  angle CFE is determined as 29⁰.

What is the measure of angle CFE?

The measure of angle CFE is calculated by applying the following formula.

The intersecting chord theorem, also known as the intersecting chords inside and outside theorem, is a geometric property that applies to chords (line segments that connect two points on the circumference of a circle) that intersect inside or outside a circle.

Based on the angle of intersecting chord theorem, we will have the following equation.

m∠CFE = ¹/₂( arc angle CE )

arc CE = 360 - 302 (sum of angles in a circle)

arc CE = 58⁰

m∠CFE = ¹/₂ x 58

m∠CFE = 29⁰

Learn more about chord angles here: brainly.com/question/23732231

#SPJ1

The sum of a number and three is no more than eight

Answers

Answer:5

Step-by-step explanation: hope this helps

Elliot read a report from a previous year saying that 6 % 6%6, percent of adults in his city biked to work. He wanted to test whether this had changed, so he took a random sample of 240 240240 adults in his city to test H 0 : p = 0.06 H 0 ​ :p=0.06H, start subscript, 0, end subscript, colon, p, equals, 0, point, 06 versus H a : p ≠ 0.06 H a ​ :p  ​ =0.06H, start subscript, start text, a, end text, end subscript, colon, p, does not equal, 0, point, 06, where p pp is the proportion of adults in Elliot's city that bike to work. The sample results showed 21 2121 adults who biked to work, and the corresponding test statistic was z ≈ 1.79 z≈1.79z, approximately equals, 1, point, 79. Assuming that the necessary conditions are met, what is the approximate P-value for Elliot's significance test?

Answers

Answer: To find the P-value, we first need to determine the direction of the alternative hypothesis. Since the alternative hypothesis is two-tailed, we need to divide the significance level (α) by 2 before finding the critical values and the rejection region. Therefore, we have:

H0: p = 0.06

Ha: p ≠ 0.06

α = 0.05/2 = 0.025

The test statistic is z = 1.79, which represents the number of standard errors that the sample proportion is from the hypothesized population proportion under the null hypothesis. We can find the P-value by looking up the area in the standard normal distribution table beyond the test statistic in both tails:

P-value = P(Z ≤ -1.79 or Z ≥ 1.79)

= P(Z ≤ -1.79) + P(Z ≥ 1.79)

= 0.0364 + 0.0364

= 0.0728

Therefore, the approximate P-value for Elliot's significance test is 0.0728. Since the P-value is greater than the significance level of 0.05, we fail to reject the null hypothesis and conclude that there is not enough evidence to suggest that the proportion of adults who bike to work in Elliot's city has changed significantly from the previous year.

Step-by-step explanation:

Choose scales for the coordinate plane shown so that you can graph the points J (5, 50), K(3, 50), L (4, -40), M (-2, 40), and N (-5, -10). on the x-axis use a scale of ____ units for each grid square. on the y-axis use a scale of ____ units of each grid square. complete the explanation for using these scales for each axis. the x-coordinates range from ____ to ____, and the y-coordinates range from ____ to ____.

Answers

On the x-axis use a scale of 2 units for each grid square. on the y-axis use a scale of 10 units of each grid square. The x-coordinates range from -5 to 5. y-coordinates range from -40 to 50.

What is coordinate plane?

In mathematics, points and functions are represented and graphed on the coordinate plane, commonly known as the Cartesian plane. The x-axis and y-axis are two perpendicular number lines that meet at the origin to form the plane (0,0).

On the coordinate plane, points are denoted by ordered pairs (x, y), where x denotes the point's separation from the y-axis and y denotes its separation from the x-axis.

For the given coordinates the range of x and y coordinates are:

x-coordinates range from -5 to 5

y-coordinates range from -40 to 50.

Thus, we use a scale of 2 on the x-axis and 10 units on the y axis.

Learn more about coordinate plane here:

https://brainly.com/question/24134413

#SPJ1

Find the area of a rectangle with length 4.3 cm and breadth 3.9 cm

Answers

Answer:

16.77 cm²

Step-by-step explanation:

Area of rectangle = L x W

L = 4.3 cm

Width = 3.9 cm

Let' solve

4.3 x 3.9 = 16.77 cm²

So, the area of the rectangle is 16.77 cm²

7. Can the following be the lengths of the sides of a triangle?
a. 20 cm, 40 cm, 50 cm
b. 20 cm, 40 cm, 60 cm
c. 41 cm, 250 mm, 12 cm​

Answers

Answer:

B

Step-by-step explanation:

THE TWO SMALLER SIDES = THE LARGER SIDE HENCE ITS A TRIANGLE

Luisa has great news, she communicates it to 3 people. Each of them tells 3 more. This is how a chain is set up, since each new person who learns the news communicates it to three others. It takes one person approximately 10 minutes to communicate the news to three others. If an hour has passed, how many people have heard the news?

Answers

Answer:

The amount of persons who have heard in 30mins would be:

45 persons

James recorded the prices of 10 brands of cereal. The prices are: $3.50, $2.30, $2.50, $3.90, $2.90, $2.40, $1.90, $2.50, $3.20, $2.70. What is the median cost of these boxes of cereal?

Answers

Answer:

To find the median cost of these boxes of cereal, we need to order the prices from least to greatest:

$1.90, $2.30, $2.40, $2.50, $2.50, $2.70, $2.90, $3.20, $3.50, $3.90

There are 10 numbers, so the median is the average of the 5th and 6th numbers:

Median = ($2.50 + $2.70) / 2 = $2.60

Therefore, the median cost of these boxes of cereal is $2.60.

Answer:

What is the median cost of these boxes of cereal?

$3.50, $2.30, $2.50, $3.90, $2.90, $2.40, $1.90, $2.50, $3.20, $2.70

Let's put it in order first:

$1.90, $2.30, $2.40, $2.50, $2.50, $2.70, $2.90, $3.20, $3.50, $3.90

Now find the middle number (s):

$1.90, $2.30, $2.40, $2.50, $2.50, $2.70, $2.90, $3.20, $3.50, $3.90

In this case, there are 2 middle numbers so do this:

$2.50 + $2.70

= $5.20

5.20 ÷ 2

= $2.60

So the median cost of those cereal boxes would be $2.60.

Step-by-step explanation:

You're welcome.

The table contains some points on the graph
of an exponential function. Based on the
table, which function represents the same
relationship?

Answers

Answer:  1st one

Step-by-step explanation:

Help pleasee
The half-life of Palladium-100 is 4 days. After 16 days a sample of Palladium-100 has been reduced to a mass of 2 mg.

What was the initial mass (in mg) of the sample? --------------


What is the mass 7 weeks after the start?-------------

Answers

The initial mass (in mg) of the sample is  32 mg

The mass 7 weeks after the start is 0.198 mg

What is exponential decay:

Exponential decay is a process in which a quantity decreases at a rate proportional to its value. This means that the larger the quantity, the faster it will decrease.

Exponential decay is modeled by the following formula:

[tex]A = A_{0} \times (1/2)^{\frac{t}{T} }[/tex]

where A is the amount after time t, A₀ is the initial amount,

T is the half-life, and t is the time elapsed

Here we have

The half-life of Palladium-100 is 4 days. After 16 days a sample of Palladium-100 has been reduced to a mass of 2 mg.

For the first question, we know that after 16 days, the amount is 2 mg, and the half-life is 4 days. We can plug these values into the formula:

[tex]2 = A_{0} \times (1/2)^{\frac{16}{4}}[/tex]

Simplifying, we get:

2 = A₀ × (1/2)⁴

2 = A₀ × (1/16)

A₀ = 2 × 16

A₀ = 32 mg

So the initial mass of the sample was 32 mg.

For the second question, we need to find the mass after 7 weeks, or 49 days. We can use the same formula, but with t = 49:

[tex]A = 32 \times (1/2)^{\frac{49}{4} }[/tex]

A ≈ 0.198 mg

Therefore,

The initial mass (in mg) of the sample is  32 mg

The mass 7 weeks after the start is 0.198 mg

Learn more about Exponential decay at

https://brainly.com/question/2193799

#SPJ1

Jeremy and Aksa were finding the volume of this prism. They agreed that 4 layers can be added together to find the volume. Jeremy says that he can see on the end of the prism that each layer will have 16 cubes in it. Aksa says that each layer has 24 cubes in it. Who is right? Explain your answer.

Answers

Both Jeremy and Aksa could be right, depending on how they are counting the cubes for finding the volume of a prism.

What is a prism?

A prism is a three-dimensional solid shape with a constant cross-section, usually a polygon. It is characterized by two identical end faces, parallel and congruent bases, and straight lateral faces connecting the bases.

According to the given information:

Both Jeremy and Aksa could be right, depending on how they are counting the cubes.

If the prism has a square base with 4 cubes along each side, then there would be 16 cubes in each layer. If there are 4 layers, then the total volume would be 16 x 4 = 64 cubic units.

However, if the prism has a rectangular base with 6 cubes along one side and 4 cubes along the other side, then there would be 24 cubes in each layer. If there are 4 layers, then the total volume would be 24 x 4 = 96 cubic units.

Therefore, the answer depends on the shape of the prism and how the layers are counted.

To know more about Prism visit:

https://brainly.com/question/23963432

#SPJ1

name three solution for h > 8

Answers

Answer:

all numbers greater than 8

Step-by-step explanation:

simply pick any number greater than 8, since the > symbol is in front of 8. By the way, you cannot pick 8. :) I hope this helps!

It's in the picture !

Answers

The position of  √17 between 4 and 5, which is nearer 5, and the location of 6, which is between 2 and 3, which is nearer 2.

What is Number line?

A visual representation of real numbers in a linear manner is a number line. It is a straight line that is divided into intervals or segments, each of which stands for a distinct real number value.

Number lines can be vertical or horizontally oriented, and they can also have a positive or negative orientation. Positive numbers often extend to the right or up, whereas negative numbers typically extend to the left or down. The origin of the number line, or zero, is typically situated in the middle of the line.

The number line is a crucial tool in mathematics since it offers a means of representing numerical relationships visually and carrying out fundamental arithmetic operations. For instance, positive and negative motions along a number line can be used to depict addition and subtraction, respectively, with positive movements denoting addition and negative movements denoting subtraction. On a number line, multiplication and division can alternatively be shown as repeated addition or subtraction.

If we have a number line with the proper markings, we may indicate where the square roots of 17 and 6 are located as follows:

To begin, determine the approximate square root values:

√17 is between 4 and 5, since 4² = 16 and 5² = 25.

√6 is between 2 and 3, since 2² = 4 and 3² = 9.

Place the number √17 closer to 5, between 4 and 5.

Place the number √6 between the numbers 2 and 3, closer to 2.

Since 6 and 17 are not integers and their values are not evenly spaced, it should be noted that their markings will not be evenly spaced on the number line.

To know more about Number Line, visit:

https://brainly.com/question/24644930

#SPJ1

Number line attached below,

simplify the expression ^5√-32x^5y^30

Answers

Sure!

The fifth root of -32x^5y^30 can be simplified as follows:

First, we can write -32 as (-2)^5.

Next, we can rewrite the expression as follows:

^5√-32x^5y^30 = ^5√((-2)^5 * x^5 * y^30)

Now we can split the root:

^5√((-2)^5 * x^5 * y^30) = (^5√(-2)^5) * (^5√x^5) * (^5√y^30)

Simplifying each part separately:

^5√(-2)^5 = -2

^5√x^5 = x

^5√y^30 = y^6

So the simplified expression is:

-2xy^6

Explain what went wrong.
Please solve both.
75 points

Answers

It’s the way they add all of the problems up

The owner of a sports complex wants to carpet a hallway connecting to buildings. The carpet costs $1.50 per square foot. How much does it cost to carpet the hallway?

Answers

Answer:

$526.75 i believe

Step-by-step explanation:

What is the range of the relation whose graph is shown?

Answers

The range of the relation whose graph is shown include the following: C. -1 ≤ y ≤ 1.

What is a domain?

In Mathematics and Geometry, a domain is the set of all real numbers for which a particular function is defined.

Additionally, the vertical extent of any graph of a function represents all range values and they are always read and written from smaller to larger numerical values, and from the bottom of the graph to the top.

By critically observing the graph shown in the image attached above, we can reasonably and logically deduce the following domain and range:

Domain = {-4, 4} or -4 ≤ x ≤ 4.

Range = {-1, 1} or -1 ≤ y ≤ 1.

Read more on domain here: brainly.com/question/17440903

#SPJ1

How to find the circumference of a circle





Answers

Answer: To find the circumference of a circle, there are 2 formulas.

Step-by-step explanation:

The first formula is 2πr and the second formula is πd when solving any math problem. You can use either one but you would use which one is the most helpful to answer your question. Hope this helps!

6. You have a device that monitors the sound level of a conversation
located 1 meter away. The results are shown in the graph.
a. Describe the relationship of the sound level as
a function of time.

Answers

At first Sound level increases then for an instant it becomes constant then in decreases for a while then again becomes constant for an instant then again starts to increase .

What is sound level of a conversation?

A decibel (dB) is a unit used to measure sound. A motorbike engine operating is around 95 dB louder than a whisper (30 dB louder than typical conversation).

                          Your hearing may begin to deteriorate if exposed to noise levels exceeding 70 dB for an extended length of time. Your ears might suffer instant damage from loud noises exceeding 120 dB.

The relationship of sound level w.r.t time can be described as ,

a) At first Sound level increases then for an instant it becomes constant then in decreases for a while then again becomes constant for an instant then again starts to increase .

b) At both the instances the sound level is increasing as the slope of the tangent drawn to the curve at those points is positive but absolute value of sound level at instance (1) is clearly lower than the value of sound level at instance (3).

Learn more about sound level of a conversation

brainly.com/question/31017529

#SPJ1

Jeff surveyed all the students at his school and reported the fraction of students who liked each flavor of sherbet.

Six-sevenths liked cherry.
One-ninth liked grape.
Three-fifths liked orange.
Four-elevenths liked strawberry.

Which list has the flavors in order from least liked to most liked?

A. grape, strawberry, orange, cherry
B. grape, orange, strawberry, cherry
C. orange, cherry, grape, strawberry
D. cherry, grape, orange. strawberry

Answers

We can compare the fractions to determine which flavors are liked the least and which are liked the most. The smallest fraction is 1/9, followed by 4/11, then 3/5, and the largest fraction is 6/7. Therefore, the list that has the flavors in order from least liked to most liked.

Answer is : A

An economy is operating with output $400 billion above its natural level, and fiscal policymakers want to close this expansionary gap. The central bank agrees to adjust the money supply to hold the interest rate constant, so there is no crowding out. The marginal propensity to consume is 3/4, and the price level is completely fixed in the short run.

to close the expansionary gap, the government would need to increase or decrease spending by $ billion.

Answers

Hence, to close the expansionary gap, the government would need to decrease spending by $100 billion.

To close the expansionary gap, the government would need to decrease spending by $100 billion.

The formula to calculate the change in equilibrium output due to a change in government spending is:

∆Y = (∆G / (1 - MPC))

Where:

∆Y = change in equilibrium output

∆G = change in government spending

MPC = marginal propensity to consume

Here, the output is $400 billion above its natural level, and the central bank agrees to adjust the money supply to hold the interest rate constant, so there is no crowding out. Therefore, we can assume that the change in government spending (∆G) would have a one-to-one effect on the equilibrium output (∆Y).

Given that MPC = 3/4, we can plug in the values into the formula:

400 = (∆G / (1 - 3/4))

∆G = (1 - 3/4) * 400

∆G = (1/4) * 400

∆G = 100

Hence, to close the expansionary gap, the government would need to decrease spending by $100 billion.

To know more about interest rate visit:

https://brainly.com/question/14445709

#SPJ1

Other Questions
QUESTION 25 1 points According to Perloff (2014), p. 453, a study of the US airline industry in early 2000's identified a number for structures for different routes. Those routes that had a Cournot market structure with three firms: Reference: Perloff, J. (2014). Microeconomics. 6th Edition. Chapter 13: Oligopolistic and Monopolistic Competition. Pearson (An electronic copy of this book chapter is available in the unit Reading List, which can be found on the right panel of the unit Blackboard site). a. Charged a price 80% higher than the marginal cost on average. O b. Charged a price 130% higher than the marginal cost on average. Oc Charged a price 30% higher than the marginal cost on average. O d.Charged a price 7 times higher than the marginal cost on averageQUESTION 26 1 points Save A According to Perloff (2014). Table 3.2. when the number of firms increases in a Cournot market structure: Reference: Perioft). (2014). Microeconomics. 6th Edition Chapter 13: Oligopolistic and Monopolistic Competition Pearson (An electronic copy of this book chapter is available in the unit Reading List which can be found on the right panel of the unit Blackboard site) a. The price decreases and the market output level decreases, and hence the deadweight loss should approach zero. b. The price approaches the marginal cost and hence the deadweight loss should approach zero. The price decreases and the market output increases, and it is not possible to tell whether the market deadweight loss cel Sore and submit to serve and submit Chick Save All Answers to save all answers, A flaw or weakness in a systems design,implementation, or operation and management that could be exploited to violatethe systems security policy is a(n) __________..a) Vulnerabilityb) Malwarec) Firewalld) Encryption what technological development made the large-scale cultivation of short-staple cotton possible? multiple choice question. automatic reaper steel plow cotton gin When is genetic drift a major factor in evolution? A. When there is sexual selection B. When there is low gene flow C. When there is high selective pressure D. When there is female choice Please help,, my teacher didnt show me how to do ths Find an equation of a line whose graph intersects the graph of the parabola y=x2 at (a) two points, (b) one point, and (c) no point. (There is more than one correct answer for each.) two young actors relocated to london and worked hard to improve their diction and accent so they could be successful on stage in theaters around the world. but whenever they went home to visit their families, after a couple days they could hear each other's old new jersey accent loud and clear. what are the actors experiencing on these visits to their hometown? choose the statements that apply to transpiration in the cohesion-tension theory. Check All That ApplyThe polar nature of water allows for cohesion.Water pressure builds in roots to drive water up the plant.Exposed water in leaves evaporates into drier sir.Water forms a continuous stream from leaf into the rootDuring transpiration, the plant is cooled as water and heat is lost. when should a hot site be used as a recovery strategy? when the organization's recovery point objective is high when the organization's disaster downtime tolerance is low when the organization's recovery time objective is high when the organization's maximum tolerable downtime is long A factory uses 4 5/6 barrels of raisins in each batch of granola bars. Yesterday, the factory used 9 2/3 barrels of raisins. How many batches of granola bars did the factory make yesterday? International money and capital markets Globalization has led to the progressive integration of capital markets around the world, allowing worldwide investors and corporations to trade with or in other countries. The following table provides descriptions of certain transactions or situations. Based on your understanding of international capital markets and international money, choose the best term to match each description. This type of bond Is issued by a European company, denominated in U.S. dollars, and sold to investors in Japan. This is the bed interest rate on a Eurodollar deposit in Europe. Deposits are made in U S. dollars outside the United States. A European pharmaceutical company invests in a manufacturing facility in India. why were chicano sociologist pushing back against the cult of objectivity, the free value ethic, and the norm of universalism in the social stances according to mary frances berry, how have most african americans viewed the meaning of the constitution? last word does leverage increase the total size of the gain or loss from an investment, or just the percentage rate of return on the part of the investment amount that was not borrowed? how would lowering leverage make the financial system more stable? if an individual with the genotype aa produces gametes, what is the probability that a gamete will receive the a allele? the members of the constitutional convention felt that, if the judicial branch was involved in any aspect of law-making, they would be four descriptive terms describing possible experience a person may have monitoring the probability or impact of the risk event to assure benefits are realized is called Name the initial and final products of photosynthesis? What purpose does the prologue serve in the text